Statistics - Poisson distribution question.

Click For Summary
The discussion revolves around a statistics problem involving the Poisson distribution, specifically calculating the probability of receiving a phone call after a certain time period. The operator receives calls at an average rate of 20 calls per hour, and the question focuses on the probability of waiting at least one more minute after no calls were received in the first three minutes. The proposed solution uses the formula P(T≥1) = 1 - FT(1) = e-1/3, leading to confusion about why the complement of receiving a call between the 3rd and 4th minutes isn't used instead. Clarification is provided that the correct approach is to calculate P{T > 4 | T > 3}, acknowledging the condition that no calls were received in the first three minutes. The explanation resolves the initial confusion regarding the application of the Poisson process.
peripatein
Messages
868
Reaction score
0
Hi,

Homework Statement


I am somewhat perplexed by the proposed solution to the following Statistics problem and was hoping one of you might be willing to help me settle this:
An operator receives phone calls between 8AM and 4PM at an average rate of 20 calls/hour. No call was received during the first 3 minutes. What is the probability that we shall have to wait at least one more minute before the first call is received?

Homework Equations


The Attempt at a Solution


Now, the book states it ought to be P(T≥1) = 1 - FT(1) = e-1/3.
My question is, why should it not have been 1 - [F(4) - F(3)], i.e. the complement of a call received between the 3rd and 4th minutes?
I'd appreciate it if any of you could explain this to me.
 
Physics news on Phys.org
peripatein said:
Hi,

Homework Statement


I am somewhat perplexed by the proposed solution to the following Statistics problem and was hoping one of you might be willing to help me settle this:
An operator receives phone calls between 8AM and 4PM at an average rate of 20 calls/hour. No call was received during the first 3 minutes. What is the probability that we shall have to wait at least one more minute before the first call is received?


Homework Equations





The Attempt at a Solution


Now, the book states it ought to be P(T≥1) = 1 - FT(1) = e-1/3.
My question is, why should it not have been 1 - [F(4) - F(3)], i.e. the complement of a call received between the 3rd and 4th minutes?
I'd appreciate it if any of you could explain this to me.

If T is the time of the first arrival, you want P{T > 4 | T > 3}, because you are told that the event {T > 3} occurred.
 
Thank you, Ray! It's clearer now :-).
 
Question: A clock's minute hand has length 4 and its hour hand has length 3. What is the distance between the tips at the moment when it is increasing most rapidly?(Putnam Exam Question) Answer: Making assumption that both the hands moves at constant angular velocities, the answer is ## \sqrt{7} .## But don't you think this assumption is somewhat doubtful and wrong?

Similar threads

Replies
2
Views
1K
  • · Replies 1 ·
Replies
1
Views
2K
  • · Replies 3 ·
Replies
3
Views
2K
  • · Replies 10 ·
Replies
10
Views
2K
  • · Replies 30 ·
2
Replies
30
Views
5K
  • · Replies 32 ·
2
Replies
32
Views
3K
Replies
2
Views
1K
Replies
56
Views
5K
  • · Replies 4 ·
Replies
4
Views
2K
Replies
1
Views
1K